Having trouble with this multiple integral question

  • Thread starter Thread starter russia123
  • Start date Start date
  • Tags Tags
    Integral Multiple
Click For Summary
The discussion revolves around solving a multiple integral problem involving the area of a triangle defined by a point P in three-dimensional space and its projection P' onto the xy-plane. The function f(x,y,z) is established as z*sqrt(x^2+y^2)/2, and the problem requires converting to polar coordinates, leading to a triple integral. The user encounters issues with the bounds for the integral, specifically questioning why the result is zero despite the bounds for theta being from 0 to 2pi. Clarifications indicate that the integral from 0 to 2pi should yield a non-zero value, emphasizing the need to reassess the integration process. The conversation highlights the importance of correctly setting up the integral and understanding the geometric interpretation of the triangle's area.
russia123
Messages
5
Reaction score
0

Homework Statement



Given a point P(x,y,z) in a three-dimensional space, let P' denote the projection of P onto the xy-plane and let O denote the origin of the coordinates, and define f(x,y,z) as the area of the triangle. Compute:

Integral of the Integral of the Integral of f(x,y,z) under E

Where E is the portion of the solid cylinder x^2+y^2=y lying between the horizontal planes z = 0 and z = 1.

2. The attempt at a solution

Okay, now I've gotten the equation of the area of the triangle: z*sqrt(x^2+y^2)/2, and then I realized that I need to change this to polar coordinates, hence:

sqrt(x^2+y^2) = r, under the assumption that r is always positive.

Due to this we get the triple integral of z*r/2*r*dr*dtheta*dz = the triple integral of z*r^2/2*dr*dtheta*dz.

Now, I've gotten to the point where my bounds for dr are 0 to sin(theta), dtheta are 0 to 2pi and z are 0 to 1, but for some reason I come out to 0. This comes from my bounds for theta being 0 to 2pi and I don't understand why this is wrong.
 
Physics news on Phys.org
f(x,y,z) is the area of what triangle?

Looking at your last integral, the integral from 0 to 2pi of dtheta is just 2pi, not 0. There are no trigonometric functions or anything that cancels out your integral
 
It is the area of the triangle formed by O, P' and P.

Well, what happens, when I try to solve it is that I need to take the integral of z*sin(theta)^3/6 with respect to theta from 0 to 2pi and that goes to 0 for me.
 
Question: A clock's minute hand has length 4 and its hour hand has length 3. What is the distance between the tips at the moment when it is increasing most rapidly?(Putnam Exam Question) Answer: Making assumption that both the hands moves at constant angular velocities, the answer is ## \sqrt{7} .## But don't you think this assumption is somewhat doubtful and wrong?

Similar threads

  • · Replies 2 ·
Replies
2
Views
2K
  • · Replies 1 ·
Replies
1
Views
2K
  • · Replies 9 ·
Replies
9
Views
2K
  • · Replies 19 ·
Replies
19
Views
2K
  • · Replies 4 ·
Replies
4
Views
2K
  • · Replies 3 ·
Replies
3
Views
1K
  • · Replies 14 ·
Replies
14
Views
2K
Replies
6
Views
2K
  • · Replies 34 ·
2
Replies
34
Views
3K
  • · Replies 6 ·
Replies
6
Views
2K